Difference between revisions of "2011 AIME I Problems/Problem 14"

(Solution 2)
m
 
(29 intermediate revisions by 14 users not shown)
Line 2: Line 2:
 
Let <math>A_1 A_2 A_3 A_4 A_5 A_6 A_7 A_8</math> be a regular octagon.  Let <math>M_1</math>, <math>M_3</math>, <math>M_5</math>, and <math>M_7</math> be the midpoints of sides <math>\overline{A_1 A_2}</math>, <math>\overline{A_3 A_4}</math>, <math>\overline{A_5 A_6}</math>, and <math>\overline{A_7 A_8}</math>, respectively.  For <math>i = 1, 3, 5, 7</math>, ray <math>R_i</math> is constructed from <math>M_i</math> towards the interior of the octagon such that <math>R_1 \perp R_3</math>, <math>R_3 \perp R_5</math>, <math>R_5 \perp R_7</math>, and <math>R_7 \perp R_1</math>.  Pairs of rays <math>R_1</math> and <math>R_3</math>, <math>R_3</math> and <math>R_5</math>, <math>R_5</math> and <math>R_7</math>, and <math>R_7</math> and <math>R_1</math> meet at <math>B_1</math>, <math>B_3</math>, <math>B_5</math>, <math>B_7</math> respectively.  If <math>B_1 B_3 = A_1 A_2</math>, then <math>\cos 2 \angle A_3 M_3 B_1</math> can be written in the form <math>m - \sqrt{n}</math>, where <math>m</math> and <math>n</math> are positive integers.  Find <math>m + n</math>.
 
Let <math>A_1 A_2 A_3 A_4 A_5 A_6 A_7 A_8</math> be a regular octagon.  Let <math>M_1</math>, <math>M_3</math>, <math>M_5</math>, and <math>M_7</math> be the midpoints of sides <math>\overline{A_1 A_2}</math>, <math>\overline{A_3 A_4}</math>, <math>\overline{A_5 A_6}</math>, and <math>\overline{A_7 A_8}</math>, respectively.  For <math>i = 1, 3, 5, 7</math>, ray <math>R_i</math> is constructed from <math>M_i</math> towards the interior of the octagon such that <math>R_1 \perp R_3</math>, <math>R_3 \perp R_5</math>, <math>R_5 \perp R_7</math>, and <math>R_7 \perp R_1</math>.  Pairs of rays <math>R_1</math> and <math>R_3</math>, <math>R_3</math> and <math>R_5</math>, <math>R_5</math> and <math>R_7</math>, and <math>R_7</math> and <math>R_1</math> meet at <math>B_1</math>, <math>B_3</math>, <math>B_5</math>, <math>B_7</math> respectively.  If <math>B_1 B_3 = A_1 A_2</math>, then <math>\cos 2 \angle A_3 M_3 B_1</math> can be written in the form <math>m - \sqrt{n}</math>, where <math>m</math> and <math>n</math> are positive integers.  Find <math>m + n</math>.
  
== Solution ==
+
 
 +
==Diagram==
 
<asy>
 
<asy>
size(200);
+
size(250);
defaultpen(linewidth(0.8));
+
pair A,B,C,D,E,F,G,H,M,N,O,O2,P,W,X,Y,Z;
real dif = 45;
+
A=(-76.537,184.776);
pair A1=dir(22.5 + 3*dif)*15,A2=dir(22.5 + 2*dif)*15,A3=dir(22.5 + dif)*15,A4=dir(22.5)*15,A5=dir(22.5 + 7*dif)*15,A6=dir(22.5 + 6*dif)*15,A7=dir(22.5 + 5*dif)*15,A8=dir(22.5 + 4*dif)*15;
+
B=(76.537,184.776);
pair M1=(A1+A2)/2,M3=(A3+A4)/2,M5=(A5+A6)/2,M7=(A7+A8)/2;
+
C=(184.776,76.537);
pair B1=extension(M1,(A4.x-1,A4.y-1),M3,(A6.x-1,A6.y+1)),B3=extension(M3,(A6.x-1,A6.y+1),M5,(A8.x+1,A8.y+1)),B5=extension(M5,(A8.x+1,A8.y+1),M7,(A2.x+1,A2.y-1)),B7=extension(M7,(A2.x+1,A2.y-1),M1,(A4.x-1,A4.y-1));
+
D=(184.776,-76.537);
draw(M1--B1^^M3--B3^^M5--B5^^M7--B7);
+
E=(76.537,-184.776);
draw(A1--A2--A3--A4--A5--A6--A7--A8--cycle);
+
F=(-76.537,-184.776);
 +
G=(-184.776,-76.537);
 +
H=(-184.776,76.537);
 +
M=(A+B)/2;
 +
N=(C+D)/2;
 +
O=(E+F)/2;
 +
O2=(A+E)/2;
 +
P=(G+H)/2;
 +
W=(100,-41.421);
 +
X=(-41.421,-100);
 +
Y=(-100,41.421);
 +
Z=(41.421,100);
 +
draw(A--B--C--D--E--F--G--H--A);
 +
label("$A_1$",A,dir(112.5));
 +
label("$A_2$",B,dir(67.5));
 +
label("$\textcolor{blue}{A_3}$",C,dir(22.5));
 +
label("$A_4$",D,dir(337.5));
 +
label("$A_5$",E,dir(292.5));
 +
label("$A_6$",F,dir(247.5));
 +
label("$A_7$",G,dir(202.5));
 +
label("$A_8$",H,dir(152.5));
 +
label("$M_1$",M,dir(90));
 +
label("$\textcolor{blue}{M_3}$",N,dir(0));
 +
label("$M_5$",O,dir(270));
 +
label("$M_7$",P,dir(180));
 +
label("$O$",O2,dir(152.5));
 +
draw(M--W,red);
 +
draw(N--X,red);
 +
draw(O--Y,red);
 +
draw(P--Z,red);
 +
draw(O2--(W+X)/2,red);
 +
draw(O2--N,red);
 +
label("$\textcolor{blue}{B_1}$",W,dir(292.5));
 +
label("$B_2$",(W+X)/2,dir(292.5));
 +
label("$B_3$",X,dir(202.5));
 +
label("$B_5$",Y,dir(112.5));
 +
label("$B_7$",Z,dir(22.5));
 
</asy>
 
</asy>
===Solution 1===
+
All distances are to scale.
Let <math>\theta=\angle M_1 M_3 B_1</math>. Thus we have that <math>\cos 2 \angle A_3 M_3 B_1=\cos(2\theta + \frac{\pi}{2})=-\sin2\theta</math>.
+
 
 +
==Solution 1==
 +
We use coordinates. Let the octagon have side length <math>2</math> and center <math>(0, 0)</math>. Then all of its vertices have the form <math>(\pm 1, \pm\left(1+\sqrt{2}\right))</math> or <math>(\pm\left(1+\sqrt{2}\right), \pm 1)</math>.
 +
 
 +
By symmetry, <math>B_{1}B_{3}B_{5}B_{7}</math> is a square. Thus lines <math>\overleftrightarrow{B_{1}B_{3}}</math> and <math>\overleftrightarrow{B_{5}B_{7}}</math> are parallel, and its side length is the distance between these two lines. However, this is given to be the side length of the octagon, or <math>2</math>.
 +
 
 +
Suppose the common slope of the lines is <math>m</math> and let <math>m=\tan\theta</math>. Then, we want to find <cmath>\cos 2\left(90-\theta\right)=2\cos^{2}\left(90-\theta\right)-1=2\sin^{2}\theta-1.</cmath>
 +
 
 +
It can easily be seen that the equations of the lines are <cmath>\begin{align*} B_{1}B_{3}: y-mx+m\left(1+\sqrt{2}\right)=0 \\ B_{5}B_{7}: y-mx-m\left(1+\sqrt{2}\right)=0.\end{align*}</cmath> By the [https://www.cuemath.com/geometry/distance-between-two-lines/ distance between parallel lines formula], a corollary of the [https://brilliant.org/wiki/dot-product-distance-between-point-and-a-line/ point to line distance formula], the distance between these two lines is <cmath>\frac{|c_{2}-c_{1}|}{\sqrt{a^{2}+b^{2}}}=\frac{2m\left(1+\sqrt{2}\right)}{\sqrt{m^{2}+1}}.</cmath> Since we want this to equal <math>2</math>, we have <cmath>\begin{align*}\frac{2m\left(1+\sqrt{2}\right)}{\sqrt{m^{2}+1}}&=2 \\ 4m^{2}\left(3+2\sqrt{2}\right)&=4m^{2}+4 \\ \left(12+8\sqrt{2}\right)m^{2}&=4m^{2}+4 \\ \left(8+8\sqrt{2}\right)m^{2}&=4 \\ m^{2}&=\frac{4}{8+8\sqrt{2}} \\ \Rightarrow m^{2}=\tan^{2}\theta=\frac{\sin^{2}\theta}{\cos^{2}\theta}&=\frac{1}{2+2\sqrt{2}}.\end{align*}</cmath> Since <math>\sin^{2}\theta+\cos^{2}\theta=1,</math> we have <math>\sin^{2}\theta=\frac{1}{3+2\sqrt{2}}</math>. Thus <cmath>2\sin^{2}\theta-1=\frac{2}{3+2\sqrt{2}}-1=\frac{-1-2\sqrt{2}}{3+2\sqrt{2}}=\frac{\left(-1-2\sqrt{2}\right)\left(3-2\sqrt{2}\right)}{\left(3+2\sqrt{2}\right)\left(3-2\sqrt{2}\right)}=\frac{5-4\sqrt{2}}{1}=5-\sqrt{32}.</cmath> The answer is <math>\boxed{037}</math>.
 +
 
 +
 
 +
==Solution 2==
 +
 
 +
Let <math>\theta=\angle M_1 M_3 B_1</math>. Thus we have that <math>\cos 2 \angle A_3 M_3 B_1=\cos \left(2\theta + \frac{\pi}{2} \right)=-\sin2\theta</math>.
  
 
Since <math>A_1 A_2 A_3 A_4 A_5 A_6 A_7 A_8</math> is a regular octagon and <math>B_1 B_3 = A_1 A_2</math>, let <math>k=A_1 A_2 = A_2 A_3 = B_1 B_3</math>.
 
Since <math>A_1 A_2 A_3 A_4 A_5 A_6 A_7 A_8</math> is a regular octagon and <math>B_1 B_3 = A_1 A_2</math>, let <math>k=A_1 A_2 = A_2 A_3 = B_1 B_3</math>.
Line 21: Line 71:
 
Extend <math>\overline{A_1 A_2}</math> and <math>\overline{A_3 A_4}</math> until they intersect. Denote their intersection as <math>I_1</math>. Through similar triangles & the <math>45-45-90</math> triangles formed, we find that <math>M_1 M_3=\frac{k}{2}(2+\sqrt2)</math>.
 
Extend <math>\overline{A_1 A_2}</math> and <math>\overline{A_3 A_4}</math> until they intersect. Denote their intersection as <math>I_1</math>. Through similar triangles & the <math>45-45-90</math> triangles formed, we find that <math>M_1 M_3=\frac{k}{2}(2+\sqrt2)</math>.
  
We also have that<math>\triangle M_7 B_7 M_1 =\triangle M_1 B_1 M_3</math> through ASA congruence (<math>\angle B_7 M_7 M_1 =\angle B_1 M_1 M_3</math>, <math>M_7 M_1 = M_1 M_3</math>, <math>\angle B_7 M_1 M_7 =\angle B_1 M_3 M_1</math>). Therefore, we may let <math>n=M_1 B_7 = M_3 B_1</math>.
+
We also have that <math>\triangle M_7 B_7 M_1 =\triangle M_1 B_1 M_3</math> through ASA congruence (<math>\angle B_7 M_7 M_1 =\angle B_1 M_1 M_3</math>, <math>M_7 M_1 = M_1 M_3</math>, <math>\angle B_7 M_1 M_7 =\angle B_1 M_3 M_1</math>). Therefore, we may let <math>n=M_1 B_7 = M_3 B_1</math>.
  
Thus, we have that <math>\sin\theta=\frac{n+k}{\frac{k}{2}(2+\sqrt2)}</math> and that <math>\cos\theta=\frac{n}{\frac{k}{2}(2+\sqrt2)}</math>. Therefore <math>\sin\theta-\cos\theta=\frac{k}{\frac{k}{2}(2+\sqrt2)}=\frac{2}{2+\sqrt2}=2-\sqrt2</math>.
+
Thus, we have that <math>\sin\theta=\frac{n-k}{\frac{k}{2}(2+\sqrt2)}</math> and that <math>\cos\theta=\frac{n}{\frac{k}{2}(2+\sqrt2)}</math>. Therefore <math>\cos\theta-\sin\theta=\frac{k}{\frac{k}{2}(2+\sqrt2)}=\frac{2}{2+\sqrt2}=2-\sqrt2</math>.
  
 
Squaring gives that <math>\sin^2\theta - 2\sin\theta\cos\theta + \cos^2\theta = 6-4\sqrt2</math> and consequently that <math>-2\sin\theta\cos\theta = 5-4\sqrt2 = -\sin2\theta</math> through the identities <math>\sin^2\theta + \cos^2\theta = 1</math> and <math>\sin2\theta = 2\sin\theta\cos\theta</math>.
 
Squaring gives that <math>\sin^2\theta - 2\sin\theta\cos\theta + \cos^2\theta = 6-4\sqrt2</math> and consequently that <math>-2\sin\theta\cos\theta = 5-4\sqrt2 = -\sin2\theta</math> through the identities <math>\sin^2\theta + \cos^2\theta = 1</math> and <math>\sin2\theta = 2\sin\theta\cos\theta</math>.
Line 29: Line 79:
 
Thus we have that <math>\cos 2 \angle A_3 M_3 B_1=5-4\sqrt2=5-\sqrt{32}</math>. Therefore <math>m+n=5+32=\boxed{037}</math>.
 
Thus we have that <math>\cos 2 \angle A_3 M_3 B_1=5-4\sqrt2=5-\sqrt{32}</math>. Therefore <math>m+n=5+32=\boxed{037}</math>.
  
===Solution 2===
+
==Solution 3==
Let <math>A_1A_2 = 2</math>.  Then <math>B_1</math> and <math>B_3</math> are the projections of <math>M_1</math> and <math>M_5</math> onto the line <math>B_1B_3</math>, so <math>2=B_1B_3=-M_1M_5\cos x</math>, where <math>x = \angle A_3M_3B_1</math>.  Then since <math>M_1M_5 = 2+2\sqrt{2}, \cos x = \dfrac{-2}{2+2\sqrt{2}}= 1-\sqrt{2}</math>,<math>\cos 2x = 2\cos^2 x -1 = 5 - 4\sqrt{2} = 5-\sqrt{32}</math>, and <math>m+n=\boxed{037}</math>.
+
Let <math>A_1A_2 = 2</math>.  Then <math>B_1</math> and <math>B_3</math> are the projections of <math>M_1</math> and <math>M_5</math> onto the line <math>B_1B_3</math>, so <math>2=B_1B_3=-M_1M_5\cos x</math>, where <math>x = \angle A_3M_3B_1</math>.  Then since <math>M_1M_5 = 2+2\sqrt{2}, \cos x = \dfrac{-2}{2+2\sqrt{2}}= 1-\sqrt{2}</math>, <math>\cos 2x = 2\cos^2 x -1 = 5 - 4\sqrt{2} = 5-\sqrt{32}</math>, and <math>m+n=\boxed{037}</math>.
 +
 
 +
==Solution 4==
 +
Notice that <math>R_3</math> and <math>R_7</math> are parallel (<math>B_1B_3B_5B_7</math> is a square by symmetry and since the rays are perpendicular) and <math>B_1B_3=B_3B_5=s=</math> the distance between the parallel rays. If the regular hexagon as a side length of <math>s</math>, then <math>M_3M_7</math> has a length of <math>s+s\sqrt{2}</math>. Let <math>X</math> be on <math>R_3</math> such that <math>M_7X</math> is perpendicular to <math>M_3X</math>, and <math>\phi=\angle M_7M_3X</math>. The distance between <math>R_3</math> and <math>R_7</math> is <math>s=M_7X</math>, so <math>\sin\phi=\frac{s}{s+s\sqrt{2}}=\frac{1}{1+\sqrt{2}}</math>.
 +
 
 +
Since we are considering a regular hexagon, <math>M_3</math> is directly opposite to <math>M_7</math> and <math>\angle A_3M_3B_1=90 ^\circ +\phi</math>. All that's left is to calculate <math>\cos 2\angle A_3M_3B_1=\cos^2(90^\circ+\phi)-\sin^2(90^\circ+\phi)=\sin^2\phi-\cos^2\phi</math>. By drawing a right triangle or using the Pythagorean identity, <math>\cos^2\phi=\frac{2+2\sqrt2}{3+2\sqrt2}</math> and <math>\cos 2\angle A_3M_3B_1=\frac{-1-2\sqrt2}{3+2\sqrt2}=5-4\sqrt2=5-\sqrt{32}</math>, so <math>m+n=\boxed{037}</math>.
 +
 
 +
 
 +
==Solution 5==
 +
Assume that <math>A_1A_2=1.</math>
 +
Denote the center <math>O</math>, and the midpoint of <math>B_1</math> and <math>B_3</math> as <math>B_2</math>. Then we have that<cmath>\cos\angle A_3M_3B_1=\cos(\angle A_3M_3O+\angle OM_3B_1)=-\sin(\angle OM_3B_1)=-\frac{OB_2}{OM_3}=-\frac{1/2}{1/2+\sqrt2/2}=-\frac{1}{\sqrt2+1}=1-\sqrt2.</cmath>Thus, by the cosine double-angle theorem,<cmath>\cos2\angle A_3M_3B_1=2(1-\sqrt2)^2-1=5-\sqrt{32},</cmath>so <math>m+n=\boxed{037}</math>.
 +
 
 +
 
 +
==Video Solution==
 +
[https://youtu.be/M47eLr9756A?si=b7aiOdKt5ZgnZY9h 2011 AIME I #14]
 +
 
 +
[https://mathproblemsolvingskills.wordpress.com/ MathProblemSolvingSkills.com]
 +
 
 +
 
  
==Diagram==
 
  
 
== See also ==
 
== See also ==
 
{{AIME box|year=2011|n=I|num-b=13|num-a=15}}
 
{{AIME box|year=2011|n=I|num-b=13|num-a=15}}
 +
 +
[[Category:Intermediate Geometry Problems]]
 
{{MAA Notice}}
 
{{MAA Notice}}

Latest revision as of 15:07, 2 September 2024

Problem

Let $A_1 A_2 A_3 A_4 A_5 A_6 A_7 A_8$ be a regular octagon. Let $M_1$, $M_3$, $M_5$, and $M_7$ be the midpoints of sides $\overline{A_1 A_2}$, $\overline{A_3 A_4}$, $\overline{A_5 A_6}$, and $\overline{A_7 A_8}$, respectively. For $i = 1, 3, 5, 7$, ray $R_i$ is constructed from $M_i$ towards the interior of the octagon such that $R_1 \perp R_3$, $R_3 \perp R_5$, $R_5 \perp R_7$, and $R_7 \perp R_1$. Pairs of rays $R_1$ and $R_3$, $R_3$ and $R_5$, $R_5$ and $R_7$, and $R_7$ and $R_1$ meet at $B_1$, $B_3$, $B_5$, $B_7$ respectively. If $B_1 B_3 = A_1 A_2$, then $\cos 2 \angle A_3 M_3 B_1$ can be written in the form $m - \sqrt{n}$, where $m$ and $n$ are positive integers. Find $m + n$.


Diagram

[asy] size(250); pair A,B,C,D,E,F,G,H,M,N,O,O2,P,W,X,Y,Z; A=(-76.537,184.776); B=(76.537,184.776); C=(184.776,76.537); D=(184.776,-76.537); E=(76.537,-184.776); F=(-76.537,-184.776); G=(-184.776,-76.537); H=(-184.776,76.537); M=(A+B)/2; N=(C+D)/2; O=(E+F)/2; O2=(A+E)/2; P=(G+H)/2; W=(100,-41.421); X=(-41.421,-100); Y=(-100,41.421); Z=(41.421,100); draw(A--B--C--D--E--F--G--H--A); label("$A_1$",A,dir(112.5)); label("$A_2$",B,dir(67.5)); label("$\textcolor{blue}{A_3}$",C,dir(22.5)); label("$A_4$",D,dir(337.5)); label("$A_5$",E,dir(292.5)); label("$A_6$",F,dir(247.5)); label("$A_7$",G,dir(202.5)); label("$A_8$",H,dir(152.5)); label("$M_1$",M,dir(90)); label("$\textcolor{blue}{M_3}$",N,dir(0)); label("$M_5$",O,dir(270)); label("$M_7$",P,dir(180)); label("$O$",O2,dir(152.5)); draw(M--W,red); draw(N--X,red); draw(O--Y,red); draw(P--Z,red); draw(O2--(W+X)/2,red); draw(O2--N,red); label("$\textcolor{blue}{B_1}$",W,dir(292.5)); label("$B_2$",(W+X)/2,dir(292.5)); label("$B_3$",X,dir(202.5)); label("$B_5$",Y,dir(112.5)); label("$B_7$",Z,dir(22.5)); [/asy] All distances are to scale.

Solution 1

We use coordinates. Let the octagon have side length $2$ and center $(0, 0)$. Then all of its vertices have the form $(\pm 1, \pm\left(1+\sqrt{2}\right))$ or $(\pm\left(1+\sqrt{2}\right), \pm 1)$.

By symmetry, $B_{1}B_{3}B_{5}B_{7}$ is a square. Thus lines $\overleftrightarrow{B_{1}B_{3}}$ and $\overleftrightarrow{B_{5}B_{7}}$ are parallel, and its side length is the distance between these two lines. However, this is given to be the side length of the octagon, or $2$.

Suppose the common slope of the lines is $m$ and let $m=\tan\theta$. Then, we want to find \[\cos 2\left(90-\theta\right)=2\cos^{2}\left(90-\theta\right)-1=2\sin^{2}\theta-1.\]

It can easily be seen that the equations of the lines are \begin{align*} B_{1}B_{3}: y-mx+m\left(1+\sqrt{2}\right)=0 \\ B_{5}B_{7}: y-mx-m\left(1+\sqrt{2}\right)=0.\end{align*} By the distance between parallel lines formula, a corollary of the point to line distance formula, the distance between these two lines is \[\frac{|c_{2}-c_{1}|}{\sqrt{a^{2}+b^{2}}}=\frac{2m\left(1+\sqrt{2}\right)}{\sqrt{m^{2}+1}}.\] Since we want this to equal $2$, we have \begin{align*}\frac{2m\left(1+\sqrt{2}\right)}{\sqrt{m^{2}+1}}&=2 \\ 4m^{2}\left(3+2\sqrt{2}\right)&=4m^{2}+4 \\ \left(12+8\sqrt{2}\right)m^{2}&=4m^{2}+4 \\ \left(8+8\sqrt{2}\right)m^{2}&=4 \\ m^{2}&=\frac{4}{8+8\sqrt{2}} \\ \Rightarrow m^{2}=\tan^{2}\theta=\frac{\sin^{2}\theta}{\cos^{2}\theta}&=\frac{1}{2+2\sqrt{2}}.\end{align*} Since $\sin^{2}\theta+\cos^{2}\theta=1,$ we have $\sin^{2}\theta=\frac{1}{3+2\sqrt{2}}$. Thus \[2\sin^{2}\theta-1=\frac{2}{3+2\sqrt{2}}-1=\frac{-1-2\sqrt{2}}{3+2\sqrt{2}}=\frac{\left(-1-2\sqrt{2}\right)\left(3-2\sqrt{2}\right)}{\left(3+2\sqrt{2}\right)\left(3-2\sqrt{2}\right)}=\frac{5-4\sqrt{2}}{1}=5-\sqrt{32}.\] The answer is $\boxed{037}$.


Solution 2

Let $\theta=\angle M_1 M_3 B_1$. Thus we have that $\cos 2 \angle A_3 M_3 B_1=\cos \left(2\theta + \frac{\pi}{2} \right)=-\sin2\theta$.

Since $A_1 A_2 A_3 A_4 A_5 A_6 A_7 A_8$ is a regular octagon and $B_1 B_3 = A_1 A_2$, let $k=A_1 A_2 = A_2 A_3 = B_1 B_3$.


Extend $\overline{A_1 A_2}$ and $\overline{A_3 A_4}$ until they intersect. Denote their intersection as $I_1$. Through similar triangles & the $45-45-90$ triangles formed, we find that $M_1 M_3=\frac{k}{2}(2+\sqrt2)$.

We also have that $\triangle M_7 B_7 M_1 =\triangle M_1 B_1 M_3$ through ASA congruence ($\angle B_7 M_7 M_1 =\angle B_1 M_1 M_3$, $M_7 M_1 = M_1 M_3$, $\angle B_7 M_1 M_7 =\angle B_1 M_3 M_1$). Therefore, we may let $n=M_1 B_7 = M_3 B_1$.

Thus, we have that $\sin\theta=\frac{n-k}{\frac{k}{2}(2+\sqrt2)}$ and that $\cos\theta=\frac{n}{\frac{k}{2}(2+\sqrt2)}$. Therefore $\cos\theta-\sin\theta=\frac{k}{\frac{k}{2}(2+\sqrt2)}=\frac{2}{2+\sqrt2}=2-\sqrt2$.

Squaring gives that $\sin^2\theta - 2\sin\theta\cos\theta + \cos^2\theta = 6-4\sqrt2$ and consequently that $-2\sin\theta\cos\theta = 5-4\sqrt2 = -\sin2\theta$ through the identities $\sin^2\theta + \cos^2\theta = 1$ and $\sin2\theta = 2\sin\theta\cos\theta$.

Thus we have that $\cos 2 \angle A_3 M_3 B_1=5-4\sqrt2=5-\sqrt{32}$. Therefore $m+n=5+32=\boxed{037}$.

Solution 3

Let $A_1A_2 = 2$. Then $B_1$ and $B_3$ are the projections of $M_1$ and $M_5$ onto the line $B_1B_3$, so $2=B_1B_3=-M_1M_5\cos x$, where $x = \angle A_3M_3B_1$. Then since $M_1M_5 = 2+2\sqrt{2}, \cos x = \dfrac{-2}{2+2\sqrt{2}}= 1-\sqrt{2}$, $\cos 2x = 2\cos^2 x -1 = 5 - 4\sqrt{2} = 5-\sqrt{32}$, and $m+n=\boxed{037}$.

Solution 4

Notice that $R_3$ and $R_7$ are parallel ($B_1B_3B_5B_7$ is a square by symmetry and since the rays are perpendicular) and $B_1B_3=B_3B_5=s=$ the distance between the parallel rays. If the regular hexagon as a side length of $s$, then $M_3M_7$ has a length of $s+s\sqrt{2}$. Let $X$ be on $R_3$ such that $M_7X$ is perpendicular to $M_3X$, and $\phi=\angle M_7M_3X$. The distance between $R_3$ and $R_7$ is $s=M_7X$, so $\sin\phi=\frac{s}{s+s\sqrt{2}}=\frac{1}{1+\sqrt{2}}$.

Since we are considering a regular hexagon, $M_3$ is directly opposite to $M_7$ and $\angle A_3M_3B_1=90 ^\circ +\phi$. All that's left is to calculate $\cos 2\angle A_3M_3B_1=\cos^2(90^\circ+\phi)-\sin^2(90^\circ+\phi)=\sin^2\phi-\cos^2\phi$. By drawing a right triangle or using the Pythagorean identity, $\cos^2\phi=\frac{2+2\sqrt2}{3+2\sqrt2}$ and $\cos 2\angle A_3M_3B_1=\frac{-1-2\sqrt2}{3+2\sqrt2}=5-4\sqrt2=5-\sqrt{32}$, so $m+n=\boxed{037}$.


Solution 5

Assume that $A_1A_2=1.$ Denote the center $O$, and the midpoint of $B_1$ and $B_3$ as $B_2$. Then we have that\[\cos\angle A_3M_3B_1=\cos(\angle A_3M_3O+\angle OM_3B_1)=-\sin(\angle OM_3B_1)=-\frac{OB_2}{OM_3}=-\frac{1/2}{1/2+\sqrt2/2}=-\frac{1}{\sqrt2+1}=1-\sqrt2.\]Thus, by the cosine double-angle theorem,\[\cos2\angle A_3M_3B_1=2(1-\sqrt2)^2-1=5-\sqrt{32},\]so $m+n=\boxed{037}$.


Video Solution

2011 AIME I #14

MathProblemSolvingSkills.com



See also

2011 AIME I (ProblemsAnswer KeyResources)
Preceded by
Problem 13
Followed by
Problem 15
1 2 3 4 5 6 7 8 9 10 11 12 13 14 15
All AIME Problems and Solutions

The problems on this page are copyrighted by the Mathematical Association of America's American Mathematics Competitions. AMC logo.png